Question:

A ball is thrown from the location \((x_0, y_0 ) = (0,0)\) of a horizontal playground with an initial speed \(𝑣_0\) at an angle \(\theta_0\)\(\theta_0\) from the +π‘₯-direction. The ball is to be hit by a stone, which is thrown at the same time from the location \((π‘₯_1, 𝑦_1 ) = (𝐿, 0).\) The stone is thrown at an angle \((180 βˆ’ \theta_1 )\) from the +π‘₯-direction with a suitable initial speed. For a fixed \(𝑣_0\) , when \((\theta_0 , \theta_1 ) = (45Β° , 45Β° ),\) the stone hits the ball after time \(𝑇_1\) , and when \((\theta_0 , \theta_1 ) = (60Β° , 30Β° )\), it hits the ball after time \(𝑇_2\) . In such a case,\( (\frac{𝑇1 }{𝑇2} )^2\) is ______.

Updated On: May 7, 2025
Hide Solution
collegedunia
Verified By Collegedunia

Correct Answer: 2

Solution and Explanation

To solve the problem, we need to determine the time at which the stone hits the ball, given their initial velocities and launch angles.

1. Equations of Motion:
Let the position of the ball at time $t$ be $(x_b, y_b)$, and the position of the stone be $(x_s, y_s)$. The equations for the motion of the ball and the stone are:

Ball: $x_b = v_0 \cos\theta_0 t$ and $y_b = v_0 \sin\theta_0 t - \frac{1}{2}gt^2$

Stone: $x_s = L - v_s \cos\theta_1 t$ and $y_s = v_s \sin\theta_1 t - \frac{1}{2}gt^2$

2. Conditions for the Stone to Hit the Ball:
For the stone to hit the ball, their positions must be equal at some time $t$, i.e., $x_b = x_s$ and $y_b = y_s$ at that time.

3. Solving for $t$:
From the $x$-position equation:

$$ v_0 \cos\theta_0 t = L - v_s \cos\theta_1 t \Rightarrow v_0 \cos\theta_0 t + v_s \cos\theta_1 t = L $$

From the $y$-position equation:

$$ v_0 \sin\theta_0 t - \frac{1}{2}gt^2 = v_s \sin\theta_1 t - \frac{1}{2}gt^2 \Rightarrow v_0 \sin\theta_0 t = v_s \sin\theta_1 t $$

So we have:

$$ v_s = \frac{v_0 \sin\theta_0}{\sin\theta_1} $$

4. Substituting into the $x$-equation:
Substituting $v_s = \frac{v_0 \sin\theta_0}{\sin\theta_1}$ into the $x$-equation:

$$ v_0 \cos\theta_0 t + \frac{v_0 \sin\theta_0}{\sin\theta_1} \cos\theta_1 t = L $$

Factor out $v_0$:

$$ v_0 (\cos\theta_0 + \sin\theta_0 \cot\theta_1)t = L $$

Solving for $t$, we get:

$$ t = \frac{L}{v_0 (\cos\theta_0 + \sin\theta_0 \cot\theta_1)} $$

5. Case 1: $\theta_0 = 45^\circ$, $\theta_1 = 45^\circ$:
For $\theta_0 = 45^\circ$ and $\theta_1 = 45^\circ$, we have:

$$ t = T_1 = \frac{L}{v_0 \left(\cos 45^\circ + \sin 45^\circ \cot 45^\circ \right)} = \frac{L}{v_0 \left(\frac{1}{\sqrt{2}} + \frac{1}{\sqrt{2}} (1)\right)} = \frac{L}{v_0 \frac{2}{\sqrt{2}}} = \frac{L}{v_0 \sqrt{2}} $$

6. Case 2: $\theta_0 = 60^\circ$, $\theta_1 = 30^\circ$:
For $\theta_0 = 60^\circ$ and $\theta_1 = 30^\circ$, we have:

$$ t = T_2 = \frac{L}{v_0 \left(\cos 60^\circ + \sin 60^\circ \cot 30^\circ\right)} = \frac{L}{v_0 \left(\frac{1}{2} + \frac{\sqrt{3}}{2} (\sqrt{3})\right)} = \frac{L}{v_0 \left(\frac{1}{2} + \frac{3}{2}\right)} = \frac{L}{v_0 (2)} $$

7. Ratio of Times:
Now, we compute the ratio of the times:

$$ \left(\frac{T_1}{T_2}\right)^2 = \left(\frac{\frac{L}{v_0 \sqrt{2}}}{\frac{L}{2v_0}}\right)^2 = \left(\frac{2}{\sqrt{2}}\right)^2 = (\sqrt{2})^2 = 2 $$

8. Final Answer:
The final answer is $\boxed{2}$.

Was this answer helpful?
0
1

Questions Asked in JEE Advanced exam

View More Questions